Difference between revisions of "2000 AMC 8 Problems/Problem 2"
Talkinaway (talk | contribs) (→Solution 3) |
|||
Line 5: | Line 5: | ||
<math>\text{(A)}\ -2 \qquad \text{(B)}\ -1 \qquad \text{(C)}\ 0 \qquad \text{(D)}\ 1 \qquad \text{(E)}\ 2</math> | <math>\text{(A)}\ -2 \qquad \text{(B)}\ -1 \qquad \text{(C)}\ 0 \qquad \text{(D)}\ 1 \qquad \text{(E)}\ 2</math> | ||
− | ==Solution 1== | + | ==Solution== |
+ | ===Solution 1=== | ||
The number <math>0</math> has no reciprocal, and <math>1</math> and <math>-1</math> are their own reciprocals. This leaves only <math>2</math> and <math>-2</math>. The reciprocal of <math>2</math> is <math>1/2</math>, but <math>2</math> is not less than <math>1/2</math>. The reciprocal of <math>-2</math> is <math>-1/2</math>, and <math>-2</math> is less than<math> -1/2</math>, so it is <math>\boxed{A}</math>. | The number <math>0</math> has no reciprocal, and <math>1</math> and <math>-1</math> are their own reciprocals. This leaves only <math>2</math> and <math>-2</math>. The reciprocal of <math>2</math> is <math>1/2</math>, but <math>2</math> is not less than <math>1/2</math>. The reciprocal of <math>-2</math> is <math>-1/2</math>, and <math>-2</math> is less than<math> -1/2</math>, so it is <math>\boxed{A}</math>. | ||
− | ==Solution 2== | + | ===Solution 2=== |
The statement "a number is less than its reciprocal" can be translated as <math>x < \frac{1}{x}</math>. | The statement "a number is less than its reciprocal" can be translated as <math>x < \frac{1}{x}</math>. | ||
Line 21: | Line 22: | ||
If <math>x<0</math>, the sign of the inequality must be switched. Thus, we have <math>x^2 > 1</math> when <math>x < 0</math>. This leads to <math>x < -1</math>. | If <math>x<0</math>, the sign of the inequality must be switched. Thus, we have <math>x^2 > 1</math> when <math>x < 0</math>. This leads to <math>x < -1</math>. | ||
− | Putting the solutions together, we have <math>x<-1</math> or <math>0 < x < 1</math>, or in interval notation, <math>(-\infty, -1) \cup(0, 1)</math>. The only answer in that range is <math>\boxed {-2 | + | Putting the solutions together, we have <math>x<-1</math> or <math>0 < x < 1</math>, or in interval notation, <math>(-\infty, -1) \cup(0, 1)</math>. The only answer in that range is <math>\boxed {\text{(A)}\ -2}</math> |
− | ==Solution 3== | + | ===Solution 3=== |
Starting again with <math>x < \frac{1}{x}</math>, we avoid multiplication by <math>x</math>. Instead, move everything to the left, and find a common denominator: | Starting again with <math>x < \frac{1}{x}</math>, we avoid multiplication by <math>x</math>. Instead, move everything to the left, and find a common denominator: |
Revision as of 14:11, 23 December 2012
Problem
Which of these numbers is less than its reciprocal?
Solution
Solution 1
The number has no reciprocal, and and are their own reciprocals. This leaves only and . The reciprocal of is , but is not less than . The reciprocal of is , and is less than, so it is .
Solution 2
The statement "a number is less than its reciprocal" can be translated as .
Multiplication by can be done if you do it in three parts: , , and . You have to be careful about the direction of the inequality, as you do not know the sign of .
If , the sign of the inequality remains the same. Thus, we have when . This leads to .
If , the inequality is undefined.
If , the sign of the inequality must be switched. Thus, we have when . This leads to .
Putting the solutions together, we have or , or in interval notation, . The only answer in that range is
Solution 3
Starting again with , we avoid multiplication by . Instead, move everything to the left, and find a common denominator:
Divide this expression at , , and , as those are the three points where the expression on the left will "change sign".
If , all three of those terms will be negative, and the inequality is true. Therefore, is part of our solution set.
If , the term will become positive, but the other two terms remain negative. Thus, there are no solutions in this region.
If , then both and are positive, while remains negative. Thus, the entire region is part of the solution set.
If , then all three terms are positive, and there are no solutions.
At all three "boundary points", the function is either or undefined. Therefore, the entire solution set is , and the only option in that region is , leading to .
See Also
2000 AMC 8 (Problems • Answer Key • Resources) | ||
Preceded by Problem 1 |
Followed by Problem 3 | |
1 • 2 • 3 • 4 • 5 • 6 • 7 • 8 • 9 • 10 • 11 • 12 • 13 • 14 • 15 • 16 • 17 • 18 • 19 • 20 • 21 • 22 • 23 • 24 • 25 | ||
All AJHSME/AMC 8 Problems and Solutions |